LSAT and Law School Admissions Forum

Get expert LSAT preparation and law school admissions advice from PowerScore Test Preparation.

User avatar
 Dave Killoran
PowerScore Staff
  • PowerScore Staff
  • Posts: 5852
  • Joined: Mar 25, 2011
|
#43586
Complete Question Explanation
(The complete setup for this game can be found here: lsat/viewtopic.php?t=8007)

The correct answer choice is (C)

Since the question asks you to find an unacceptable grouping of fish, and answer choice (C) contains H but not K, answer choice (C) is correct.
 Leonard
  • Posts: 2
  • Joined: Jul 26, 2018
|
#48862
Hello, i dont understand why answer choices a,b,d or e are an okay group. The last rule states that if she selects k then she must select x, so none of the answers should make sense?
 Adam Tyson
PowerScore Staff
  • PowerScore Staff
  • Posts: 5153
  • Joined: Apr 14, 2011
|
#49011
Pay close attention to every word in that question, Leonard, and you'll see that it is asking only about the acceptable groups of fish in the aquarium. The fish are GHJKL - WXYZ are plants! This question isn't asking about complete solutions to the game, but about partial solutions for just the fish.

Answer A works, because with GKL as the fish, the plants could be X and W or X and Z (K requires X, so that has to be selected, and G knocks out Y, so that cannot be selected)

Answer B works because with HJK as the fish, X and W would be the plants (K requires X, J requires W)

Answers D and E also work as the groups of fish, because they get along with each other and we can find an acceptable group of plants to join them.

Take another look and see if that makes sense, Leonard! Going forward, be careful, because every word matters!

Get the most out of your LSAT Prep Plus subscription.

Analyze and track your performance with our Testing and Analytics Package.